Answer (C) is correct . The firm will produce 9 units because at that level marginal revenue ($290) is closest to marginal cost ($285). At $290 per unit, revenue will be $2,610. At $205 per unit, total cost will be $1,845, leaving $765 of profit.
Answer (A) is incorrect because At 7?units of output, marginal costs are less than marginal revenue; a rational firm will increase production. Answer (B) is incorrect because At 8?units of output, marginal costs are less than marginal revenue; a rational firm will increase production. Answer (D) is incorrect because At 10 units of output, marginal costs are higher than marginal revenue; thus, economic profit will be less for an output of 10 units than for an output of 9 units.
|